Need expert help - Assumption

This topic has expert replies
Master | Next Rank: 500 Posts
Posts: 405
Joined: Thu Feb 10, 2011 1:44 am
Thanked: 3 times
Followed by:1 members

Need expert help - Assumption

by voodoo_child » Tue Jul 17, 2012 9:17 am
Journalist: Workers at Facsum Inc. have threatened to strike if management does not meet their demands for an immediate 5% pay raise and a paid lunch break. Further, workers are insisting that the company rehire 12 employees who were laid off for complaining about substandard wages and working conditions. It is well-known that Facsum reported that negative profits in 3 of its previous 4 quarterly earnings reports. We believe a strike is inevitable.

Identify an assumption required by the journalist's argument


The workers would likely be willing to compromise with respect to the paid lunch break.
Facsum is unwilling to negotiate with the workers.
The majority of the losses were due to a significant decline in profit margins.
The 12 laid-off employees were not laid off for any performance-related reasons.
Facsum likely does not have sufficient cash flow or cash reserves to support increased expenses.

OA - E.

Can someone explain why B) is incorrect?

Thanks

User avatar
Legendary Member
Posts: 520
Joined: Sat Apr 28, 2012 9:12 pm
Thanked: 339 times
Followed by:49 members
GMAT Score:770

by eagleeye » Tue Jul 17, 2012 10:12 am
voodoo_child wrote:Journalist: Workers at Facsum Inc. have threatened to strike if management does not meet their demands for an immediate 5% pay raise and a paid lunch break. Further, workers are insisting that the company rehire 12 employees who were laid off for complaining about substandard wages and working conditions. It is well-known that Facsum reported that negative profits in 3 of its previous 4 quarterly earnings reports. We believe a strike is inevitable.

Identify an assumption required by the journalist's argument


The workers would likely be willing to compromise with respect to the paid lunch break.
Facsum is unwilling to negotiate with the workers.
The majority of the losses were due to a significant decline in profit margins.
The 12 laid-off employees were not laid off for any performance-related reasons.
Facsum likely does not have sufficient cash flow or cash reserves to support increased expenses.

OA - E.

Can someone explain why B) is incorrect?

Thanks
B is incorrect because nowhere in the passage is it suggested that Facsum is "unwilling" to negotiate. We know that workers demands are not being met at the moment; however, that does not mean that Facsum are not willing to negotiate. They may be or they may not be. We just don't know. Hence B is not a required assumption.

Cheers!

Junior | Next Rank: 30 Posts
Posts: 26
Joined: Thu Dec 22, 2011 10:22 pm
Thanked: 4 times
Followed by:1 members

by Shalini Suresh » Tue Jul 17, 2012 10:22 am
Since the journalist has mentioned 'It is well-known that Facsum reported that negative profits in 3 of its previous 4 quarterly earnings reports' its safe to assume that Facsum is cash strapped and thats why they wouldnt be able to meet the workers' demands. So E) would be correct.

Master | Next Rank: 500 Posts
Posts: 222
Joined: Mon Oct 13, 2008 4:04 pm
Thanked: 3 times
Followed by:2 members

by venmic » Thu Jul 19, 2012 7:16 pm
Factors or issues can never be an assumption

B sounds more like a possibility

Newbie | Next Rank: 10 Posts
Posts: 5
Joined: Thu Jun 14, 2012 11:29 am
GMAT Score:680

by faltuhaiye11 » Tue Jul 24, 2012 10:53 am
please provide the source of this question ..
"D" day matter .. not Mocks...
GPREP1- 740 ; GPREP2 700 ;
MGMAT 1:650 ; MGMAT 2 :700
MGMAT 3 :710 ; MGMAT 4 :720
MGMAT 5 :730 ; MGMAT 6 :730
D day - 680

User avatar
Newbie | Next Rank: 10 Posts
Posts: 6
Joined: Mon Jul 12, 2010 6:12 am
Thanked: 1 times

by vbalhara » Tue Aug 21, 2012 2:26 am
Assumptions are of 2 types
1) protect the conclusion
2) link the related premises to form a good conclusion .

This example deals with 2 category.

B and E are the only relevant choice.


Conclusion workers will go on strike (they are demanding hike)

What can connect strike with demand? - Money :)




When left with 2 choices .. Negate each of them and see which weakens the conclusion strongly.


Company is willing to negotiate. -- Impact on conclusion -> MILD (worker may go with strike)
Company have money -- Impact on conclusion greater than above

User avatar
Senior | Next Rank: 100 Posts
Posts: 83
Joined: Sun Aug 19, 2012 12:42 am

by hjafferi » Tue Aug 21, 2012 7:25 am
Got this wrong...chose D instead of E

Thanks vbalhara for the clarification

Senior | Next Rank: 100 Posts
Posts: 57
Joined: Thu Aug 23, 2012 7:50 pm
Thanked: 4 times

by 7777 » Fri Aug 24, 2012 10:03 pm
eagleeye wrote:
voodoo_child wrote:Journalist: Workers at Facsum Inc. have threatened to strike if management does not meet their demands for an immediate 5% pay raise and a paid lunch break. Further, workers are insisting that the company rehire 12 employees who were laid off for complaining about substandard wages and working conditions. It is well-known that Facsum reported that negative profits in 3 of its previous 4 quarterly earnings reports. We believe a strike is inevitable.

Identify an assumption required by the journalist's argument


The workers would likely be willing to compromise with respect to the paid lunch break.
Facsum is unwilling to negotiate with the workers.
The majority of the losses were due to a significant decline in profit margins.
The 12 laid-off employees were not laid off for any performance-related reasons.
Facsum likely does not have sufficient cash flow or cash reserves to support increased expenses.

OA - E.

Can someone explain why B) is incorrect?

Thanks
B is incorrect because nowhere in the passage is it suggested that Facsum is "unwilling" to negotiate. We know that workers demands are not being met at the moment; however, that does not mean that Facsum are not willing to negotiate. They may be or they may not be. We just don't know. Hence B is not a required assumption.

Cheers!
negotiation is successful when both the parties are willing to negotiate. what if workers dont want to negotiate. this can't be an assumption.
negating the sentence makes it " facsum is willing to negotiate." so what?? it doesnt undermine the conclusion that strike is inevitable.